\documentclass[全部作业]{subfiles} \input{mysubpreamble} \begin{document} \setcounter{chapter}{1} \section{《随机过程》第二周作业} \setcounter{subsection}{2} \subsection{练习题\thechapter.\thesubsection} \begin{enumerate} \questionandanswerProof[1]{ 设$X$是连续型非负随机变量,证明$L_{X}(t)$在$[0,+\infty)$内是一致连续的。 }{ % 注意到$f(t)=e^{-t}$在$[0,+\infty)$内是一致连续的,即$\forall \varepsilon>0, \exists \delta_0 >0, \forall t_1,t_2 \in [0,+\infty)$,且$\left\vert t_1-t_2 \right\vert <\delta_0$,都有$\left\vert e^{-t_1}-e^{-t_2} \right\vert <\dfrac{\varepsilon}{Ee^{X}}$,所以$Ee^{X}\left\vert e^{-t_1}-e^{-t_2} \right\vert <\varepsilon$,即$\left\vert Ee^{-t_1X}-Ee^{-t_2X} \right\vert =Ee^{-t_1X}(1-e^{(t_1-t_2)X})$ 设$X$的概率密度函数为$p(x)$。使用点列式等价定义,$\forall t_{1n}, t_{2n}\in [0,+\infty)$且$\left\vert t_{1n}-t_{2n} \right\vert \to 0$ $$ \begin{aligned} &\left\vert L_{X}(t_{1n})-L_{X}(t_{2n}) \right\vert = \left\vert Ee^{-t_{1n}X}-Ee^{-t_{2n}X} \right\vert =\left\vert E(e^{-t_{1n}X}-e^{-t_{2n}X}) \right\vert \\ &=\left\vert \int_{0}^{+\infty} (e^{-t_{1n}x}-e^{-t_{2n}x})p(x) \mathrm{d}x \right\vert =\left\vert \int_{0}^{+\infty} e^{-t_{1n}x}(1-e^{(t_{1n}-t_{2n})x}) p(x)\mathrm{d}x \right\vert \\ &\leqslant \left\vert \int_{0}^{+\infty} (1-e^{(t_{1n}-t_{2n})x}) p(x)\mathrm{d}x \right\vert \\ \end{aligned} $$ 根据积分中值定理,$\exists \xi \in [0,+\infty) $,使得 $$ \left\vert \int_{0}^{+\infty} (1-e^{(t_{1n}-t_{2n})x})p(x) \mathrm{d}x \right\vert =(1-e^{(t_{1n}-t_{2n})\xi}) \int_{0}^{+\infty} p(x) \mathrm{d}x=1-e^{(t_{1n}-t_{2n})x} $$ 因为$\left\vert t_{1n}-t_{2n} \right\vert \to 0$,所以$1-e^{(t_{1n}-t_{2n})x} \to 0$,因此$\left\vert L_{X}(t_{1n}-t_{2n}) \right\vert \to 0$,从而$L_{X}(t)$在$[0,+\infty)$内是一致收敛的。 } \questionandanswer[5]{ 设独立同分布的标准正态分布随机变量簇$W_m,m>1$与$\{ N(n),n\geqslant 1 \}$独立,其中$N(n)$服从泊松分布$P(n)$。令$Y_n=\sum_{k=1}^{N(n)}W_k$。求$Y_n$的特征函数,并证明$n \to +\infty$时$\dfrac{Y_n}{\sqrt{n}}$依分布收敛到一个服从标准正态分布的随机变量。 }{ \begin{solution} $Y_n$的特征函数为 $$ \begin{aligned} \psi_{Y_n}(t)=E e^{it Y_n}=Ee^{it \sum_{k=1}^{N(n)}W_k}=E e^{\sum_{k=1}^{N(n)}it W_k}=E \prod_{k=1}^{N(n)} e^{it W_k} \end{aligned} $$ 根据重期望公式, $$ \text{上式}=E\left( E \left(\left. \prod_{k=1}^{N(n)} e^{it W_k} \right| N(n)=p \right) \right) $$ 由于$W_k$独立同分布且服从标准正态分布,所以 $$ E\left( \left. \prod_{k=1}^{N(n)} e^{it W_k} \right|N(n)=p \right) =\prod_{k=1}^{p} E e^{it W_k}= \prod_{k=1}^{p} \psi_{W_1}(t)= \left[ \psi_{W_1}(t) \right] ^{p}=\left[ e^{-\frac{t^{2}}{2}} \right] ^{p}=e^{-\frac{p t^{2}}{2}} $$ 所以 % = e^{- n} \sum_{k=1}^{\infty} \frac{n^{k} e^{- \frac{k t^{2}}{2}}}{\Gamma(k + 1)} $$ \psi_{Y_n}(t)=E \left( e^{-\frac{t^{2}N(n)}{2}} \right) = \sum_{k=1}^{\infty}e^{- \frac{t^{2}k}{2}} \frac{n^{k}}{k!}e^{-n} % = e^{- \frac{nt^{2}}{2}} $$ \end{solution} \begin{proof} 根据特征函数的性质, $$ \psi_{\frac{Y_n}{\sqrt{n}}}(t)=\psi_{Y_n}\left( \frac{t}{\sqrt{n}} \right) % =e^{-\frac{n \left( \frac{t}{\sqrt{n}} \right) ^{2}}{2}}=e^{-\frac{t^{2}}{2}} = \sum_{k=1}^{\infty}e^{-\frac{\left( \frac{t}{\sqrt{n}} \right) ^{2} \cdot k}{2}} \frac{n^{k}}{k!}e^{-n} = e^{- n} \sum_{k=1}^{\infty} \frac{n^{k} e^{- \frac{k t^{2}}{2 n}}}{k!} $$ 当$n \to +\infty$时上式$\to e^{-\frac{t^{2}}{2}}$,即$\frac{Y_n}{\sqrt{n}}$的特征函数收敛到标准正态分布的特征函数。根据特征函数的唯一性,则$n \to +\infty$时$\frac{Y_n}{\sqrt{n}}$依分布收敛到一个服从标准正态分布的随机变量。 \end{proof} } \questionandanswerSolution[7]{ 若随机变量$X$的分布列为$P(X=n)=\frac{n}{2^{n+1}},n\geqslant 1$,求$X$的概率母函数$\phi(s)$。 }{ $$ \phi(s)=E s^{X}=\sum_{n=1}^{\infty}s^{n} \cdot \frac{n}{2^{n+1}} = \frac{s}{(s - 2)^{2}}\ , \quad s \in [-1,1] $$ } \questionandanswerSolution[8]{ 已知概率母函数$\phi(s)=s+ \frac{1}{1+\gamma}(1-s)^{1+\gamma}, \gamma\in (0,1]$,求对应的概率分布列。 }{ $$ P(X=k)=\frac{\phi^{(k)}(0)}{k!} $$ \begin{minipage}{0.6\linewidth} 对$\phi(s)$求各阶导数: $$ \phi^{(k)}(x)=\begin{cases} s+ \frac{1}{1+\gamma}(1-s)^{1+\gamma},\quad & k=0 \\ 1-(1-s)^{\gamma},\quad & k=1 \\ \displaystyle (-1)^{k}(1-s)^{\gamma-k+1} \prod_{i=0}^{k-2} (\gamma-i),\quad & k\geqslant 2 \\ \end{cases}; $$ \end{minipage} \begin{minipage}{0.4\linewidth} 所以 $$ \phi^{(k)}(0)=\begin{cases} \frac{1}{1+\gamma},\quad & k=0 \\ 0,\quad & k=1 \\ \displaystyle (-1)^{k}\prod_{i=0}^{k-2} (\gamma-i),\quad & k\geqslant 2 \\ \end{cases} $$ \end{minipage} 所以对应的概率分布列为 $$ P(X=k)=\begin{cases} \frac{1}{1+\gamma},\quad & k=0 \\ 0,\quad & k=1 \\ \displaystyle \frac{(-1)^{k}}{k!}\prod_{i=0}^{k-2}(\gamma-i) ,\quad & k\geqslant 2 \\ \end{cases} $$ } \end{enumerate} \pagebreak[4] % 不知道为什么这行如果强度改成4会导致页码问题 \subsection{练习题\thechapter.\thesubsection} \begin{enumerate} \questionandanswerProof[1]{ 若存在随机变量$X$使得随机序列$\dfrac{S_n}{n}\xlongrightarrow{a.s.}X$成立,而且非负整数值随机序列$N_n \xlongrightarrow{a.s.}\infty$成立。证明$\dfrac{S_{N_n}}{N_n}\xlongrightarrow{a.s.}X$。 }{ 对于$\forall \varepsilon>0$,由$\frac{S_n}{n} \xlongrightarrow{a.s.} X$的等价定义,$\exists A \in \Omega$满足$P(A)=0$,对$\forall \omega \in \Omega A$以及$\forall \varepsilon>0$,$\exists N=N(\varepsilon,\omega)$,使得当$n>N$时$\left\vert \frac{S_n(\omega)}{n}-X(\omega) \right\vert <\varepsilon$ 对于上述的$N$,由$N_n\xlongrightarrow{a.s.}\infty$的等价定义,$\exists A'\in \Omega$满足$P(A')=0$,对$\forall \omega' \in \Omega A$,$\exists M=M(\varepsilon,\omega)$,使得当$n>M$时$N_n(\omega')> N$,再由上述叙述可知$\left\vert \displaystyle \frac{S_{N_n(\omega')}(\omega)}{N_n(\omega')}-X(\omega) \right\vert <\varepsilon$ 因此$\displaystyle \frac{S_{N_n}}{N_n}\xlongrightarrow{a.s.}X$ } \questionandanswer[2]{ 证明:若对任意$\varepsilon>0$,存在$N$使得当$ n>N$时$P(\left\vert X_n-X \right\vert <\varepsilon)=1$,那么$X_n\xlongrightarrow{a.s.}X$。举例说明逆命题不成立。 }{ \begin{proof} 由于$P(\left\vert X_n-X \right\vert <\varepsilon)=1 \Rightarrow P(\left\vert X_n-X \right\vert \geqslant \varepsilon)=0$, 题中条件可转换为 $$ \text{对}\forall \varepsilon>0, P\left( \bigcap_{N=1} ^{\infty}\bigcup_{n=N} ^{\infty}\{ \left\vert X_n-X \right\vert \geqslant \varepsilon \}\right)=0 $$ 所以$X_n\xlongrightarrow{a.s.}X$。 \end{proof} 当$P(\left\vert X_n-X \right\vert \geqslant \varepsilon)=0$但$P(\left\vert X_n-X \right\vert <\varepsilon)\neq 1$时,逆命题不成立。 } \questionandanswerProof[4]{ 设$f(x)$在$[0,+\infty)$上连续有界,单调上升且$f(0)=0$,证明随机变量$X_n$依概率收敛于0当且仅当$\displaystyle \lim_{n \to \infty}E(f(\left\vert X_n \right\vert ))=0$。 }{ 若$X_n \xrightarrow{P} 0$,则$\forall \varepsilon>0, P(\left\vert X_n \right\vert >\varepsilon) \to 0$,由于$f(x)$的单调性,$P(f(\left\vert X_n \right\vert )>f(\varepsilon)) \to 0$。 令$\varepsilon \to 0$,则$f(\varepsilon)\to f(0)=0$,所以$P(f(\left\vert X_n \right\vert )>0)\to 0$。 因为$\left\vert X_n \right\vert \geqslant 0$,且$f(x)$单调上升且$f(0)=0$,所以$f(\left\vert X_n \right\vert )\geqslant 0$,因此$P(f(\left\vert X_n \right\vert )=0)\to 1$。 $f(\left\vert X_n \right\vert )$在0处的概率趋向于1,所以$\displaystyle \lim_{n \to \infty}E(f\left\vert X_n \right\vert )=0$。 另一方向: 若$\displaystyle \lim_{n \to \infty}E(f(\left\vert X_n \right\vert ))=0$,由于$f(\left\vert X_n \right\vert )$是非负随机变量,所以期望为0等价于在0处的概率为1,即$\displaystyle \lim_{n \to \infty}P(f(\left\vert X_n \right\vert )=0)=1$,即$P(f(\left\vert X_n \right\vert )=0)\to 1$。 由于$f(\left\vert X_n \right\vert )\geqslant 0$,所以$P(f(\left\vert X_n \right\vert )>0)\to 0$。 再根据$f(x)$的单调性,$P(\left\vert X_n \right\vert >0) \to 0$。 因此$\forall \varepsilon>0, P(\left\vert X_n \right\vert >\varepsilon) \to 0$,即$X_n$依概率收敛于0。 } \questionandanswerProof[5]{ 设$\{ X_k;k\geqslant 1 \}$为一列相互独立随机变量,而且对任意$k\geqslant 1$,$E(X_k)=0$,$\operatorname{Var}(X_k)=k$。对任意$n\geqslant 1$,令$\displaystyle Y_n=\sum_{k=1}^{n}\frac{X_k}{k}$。 证明对任意$r>\dfrac{1}{2}$,当$n \to \infty$时$\dfrac{Y_n}{n^{r}}$几乎必然收敛到0。 }{ 要证$\dfrac{Y_n}{n^{r}}\xrightarrow{a.s.}0$,即证$\forall \varepsilon>0, \displaystyle \sum_{n=1}^{\infty}P\left( \left\vert \frac{Y_n}{n^{r}}\right\vert>\varepsilon \right) <\infty $。 由马尔可夫不等式, $$ \displaystyle P\left( \left\vert \frac{Y_n}{n^{r}} \right\vert >\varepsilon \right) <\frac{E\left\vert \frac{Y_n}{n^{r}} \right\vert }{\varepsilon}=\frac{E\left\vert \displaystyle \sum_{k=1}^{n}\frac{X_k}{k} \right\vert }{n^{r}\varepsilon} $$ 由于$\operatorname{Var}(X_k)=k, std(X_k)=\sqrt{k}$且$E(X_k)=0$,所以$std(\frac{X_k}{k})=\frac{1}{\sqrt{k}}$,所以$\operatorname{Var}(\frac{X_k}{k})=\frac{1}{k}$。 $$ E\left\vert \sum_{k=1}^{n}\frac{X_k}{k} \right\vert \leqslant \left\vert \sum_{k=1}^{n}\frac{1}{\sqrt{k}} \right\vert \leqslant \left\vert \sum_{k=1}^{n}\frac{1}{\sqrt{n}} \right\vert = \frac{n}{\sqrt{n}}=\sqrt{n} $$ 所以 $$ P\left( \left\vert \frac{Y_n}{n^{r}} \right\vert >\varepsilon \right) < \frac{\sqrt{n}}{n^{r}\varepsilon}=n^{\frac{1}{2}-r}\varepsilon $$ 对$\forall r>\frac{1}{2}$,当$n \to \infty$时$n^{\frac{1}{2}-r} \to 0$。 所以$\forall \varepsilon>0, \displaystyle \sum_{n=1}^{\infty}P\left( \left\vert \frac{Y_n}{n^{r}} \right\vert >\varepsilon \right) $收敛。 因此$\forall r>\frac{1}{2}$,当$n \to \infty$时$\displaystyle \frac{Y_n}{n^{r}}$几乎必然收敛到0。 } \end{enumerate} \end{document}